K
Khách

Hãy nhập câu hỏi của bạn vào đây, nếu là tài khoản VIP, bạn sẽ được ưu tiên trả lời.

25 tháng 4 2016

t/x là ?

NV
20 tháng 3 2022

Xét đa thức bậc 8: \(P\left(x\right)=x^8+\dfrac{x^3-x}{2}\)

Ta có, \(P\left(x\right)-P\left(-x\right)=x^8+\dfrac{x^3-x}{2}-\left(-x\right)^8-\dfrac{\left(-x\right)^3-\left(-x\right)}{2}=x^3-x\)

Thay \(x=1;2;3;4\) đều thỏa mãn

\(\Rightarrow P\left(5\right)-P\left(-5\right)=5^3-5=120\)

20 tháng 3 2022

Em cám ơn thầy Lâm ạ!

NV
13 tháng 9 2021

Đặt \(H\left(x\right)=P\left(x\right)-\left(x^2+2\right)\)

\(\Rightarrow H\left(1\right)=H\left(3\right)=H\left(5\right)=0\)

\(\Rightarrow H\left(x\right)\) có 3 nghiệm 1; 3; 5

\(\Rightarrow H\left(x\right)=\left(x-1\right)\left(x-3\right)\left(x-5\right)\left(x-a\right)\)

\(\Rightarrow P\left(x\right)=H\left(x\right)+x^2+2=\left(x-1\right)\left(x-3\right)\left(x-5\right)\left(x-a\right)+x^2+2\)

\(\Rightarrow P\left(-2\right)+7P\left(6\right)=-105\left(-2-a\right)+4+2+7\left[15\left(6-a\right)+36+2\right]=1112\)

6 tháng 3 2018

Bài 1 : k bt làm

Bài 2 :

Ta có : \(\left(x-6\right).P\left(x\right)=\left(x+1\right).P\left(x-4\right)\) với mọi x

+) Với \(x=6\Leftrightarrow\left(6-6\right).P\left(6\right)=\left(6+1\right).P\left(6-4\right)\)

\(\Leftrightarrow0.P\left(6\right)=7.P\left(2\right)\)

\(\Leftrightarrow0=7.P\left(2\right)\)

\(\Leftrightarrow P\left(2\right)=0\)

\(\Leftrightarrow x=2\) là 1 nghiệm của \(P\left(x\right)\left(1\right)\)

+) Với \(x=-1\Leftrightarrow\left(-1-6\right).P\left(-1\right)=\left(-1+1\right).P\left(-1-4\right)\)

\(\Leftrightarrow\left(-7\right).P\left(-1\right)=0.P\left(-5\right)\)

\(\Leftrightarrow\left(-7\right).P\left(-1\right)=0\)

\(\Leftrightarrow P\left(-1\right)=0\)

\(\Leftrightarrow x=-1\) là 1 nghiệm của \(P\left(x\right)\) \(\left(2\right)\)

Từ \(\left(1\right)+\left(2\right)\Leftrightarrow P\left(x\right)\) có ót nhất 2 nghiệm

6 tháng 3 2018

nghiệm của đa thức xác định đa thức đó bằng 0

0 mà k bằng 0. You định làm nên cái nghịch lý ak -.-

NM
19 tháng 1 2021

Ta đi phản chứng, giả sử P(x) có thể phân tích được thành tích hai đa thức hệ số nguyên bậc lớn hơn 1.

đặt \(P\left(x\right)=Q\left(x\right).H\left(x\right)\)với bậc của Q(x) và H(x) lớn hơn 1

Ta Thấy \(Q\left(i\right).H\left(i\right)=P\left(i\right)=-1\)với i=1,2,...2020.

suy ra \(\hept{\begin{cases}Q\left(i\right)=1\\H\left(i\right)=-1\end{cases}}\)hoặc \(\hept{\begin{cases}Q\left(i\right)=-1\\H\left(i\right)=1\end{cases}}\) suy ra \(Q\left(i\right)+H\left(i\right)=0\)với i=1,2,...,2020

mà bậc của Q(x) và H(x) không vượt quá 2019 suy ra \(Q\left(x\right)+H\left(x\right)=0\Rightarrow Q\left(x\right)=-H\left(x\right)\Rightarrow P\left(x\right)=-\left(Q\left(x\right)\right)^2\)

xét hệ số đơn thức bậc cao nhất của \(P\left(x\right)\) bằng 1 

hệ số đơn thức bậc cao nhất của \(-\left(Q\left(x\right)\right)^2\) bằng -1.  Suy ra vô lý. 

Vậy P(x)  không thể phân tích thành hai đa thức hệ số nguyên có bậc lớn hơn 1.

28 tháng 3 2018

Nhiều nick nhỉ! :)

28 tháng 3 2018

ai giúp cho 10 like